Why is atheism rational?

destinyawaits
Posts: 9
Joined: 2008-06-03
User is offlineOffline
Why is atheism rational?

I'm new here and it's a bit difficult to know where to start. So I've started a new thread. Would someone please explain how the belief that we are finite physical beings is rational and how the belief that we are not is irrational. I don't belong to any church or group, I had a Godless upbringing,  but I do believe there is more to life than a brief physical existence. Science can demonstrate various cause and effect relationships and obviously this is very useful if you want to build an engine or cure a disease but as for spiritual matters all science can say is "no evidence". And of course their is no scientific evidence because science is concerned exclusively with material relationships. Subjective experience is beyond science. We cannot prove that someone has heard the same piece of music as another person or that two people have have seen the same image or colour. Much of what we think we know is assumption, habit, convention, laziness etc. Why should  "freethinking" be restricted to the belief that we are physical beings with a  restricted lifespan. This is a belief, a dogma like any other. It's a restriction on the imagination and very narrow filter for experience.     


jcgadfly
Superfan
Posts: 6791
Joined: 2006-07-18
User is offlineOffline
Paul Hakel wrote:Atheism, I

Paul Hakel wrote:

Atheism, I would argue, is not rational or irrational, but non-rational. I don't think that it's a proven fact, nor is theism, and we are to believe, which puts it in the realm of the non-rational. 

Ok.

Do you have an argument to go with this claim that non-belief is really a belief?

When you're through watching TV, do you turn it off or change it to the "OFF" channel?

"I do this real moron thing, and it's called thinking. And apparently I'm not a very good American because I like to form my own opinions."
— George Carlin


shikko
Posts: 448
Joined: 2007-05-23
User is offlineOffline
Paul Hakel wrote:Atheism, I

Paul Hakel wrote:

Atheism, I would argue, is not rational or irrational, but non-rational. I don't think that it's a proven fact, nor is theism, and we are to believe, which puts it in the realm of the non-rational. 

You could argue that, but you would lose.

Theists are the ones that have to prove their position, since they are the ones making a positive claim.  I'm surprised that you know there's a difference between irrational and non-rational, while simultaneously not understanding the "burden of proof."

Theists believe in the existence of at least one god.  Therefore, the onus is on them to find and articulate real evidence of that claim.  Until they do, their faith should be called what it is: unsubstantiated.

 

 

--
maybe if this sig is witty, someone will love me.


nigelTheBold
atheist
nigelTheBold's picture
Posts: 1868
Joined: 2008-01-25
User is offlineOffline
The non-observable as rationalism

Paul Hakel wrote:

Atheism, I would argue, is not rational or irrational, but non-rational. I don't think that it's a proven fact, nor is theism, and we are to believe, which puts it in the realm of the non-rational. 

I'd argue that you don't prove facts, you observe facts. Then you come up with explanations for those facts, and test those explanations by the predictions they make, usually by observing new facts. "Rationality" is the art of judging the correctness of an explanation by its congruence with observed facts, and its ability to predict new facts.

You can't prove anything to the point of absolute certainty. Hypothesis and theories are not "facts," they are descriptions of processes and inter-relations. As such, they are potentially incorrect. They are always contingent upon new data, new observed facts.

This doesn't exclude them from rationality in the slightest.

Not believing in magic is not only rational, it is the only logical conclusion. Belief in magic (or God, or any other supernatural being) is irrational. Not because it hasn't been proven, but because it is completely contrary to our observed facts, and our contingent (and rational) hypothesis and theories. That is: it is completely incongruent with observed reality.

Anyway, so far I've not seen a decent argument against the rationalism of atheism. A lack of belief in something that isn't observably there is perfectly rational. It should be the default position for all of the infinite things we have not observed to be there.

"Yes, I seriously believe that consciousness is a product of a natural process. I find that the neuroscientists, psychologists, and philosophers who proceed from that premise are the ones who are actually making useful contributions to our understanding of the mind." - PZ Myers


I AM GOD AS YOU
Superfan
Posts: 4793
Joined: 2007-09-29
User is offlineOffline
Yeah, there is nothing to

Yeah Bold, there is nothing to prove , all we do is look and wonder in awe ..... and try to understand as in math, as in philosophy, while Religion just fucking lies all the time .... look at them silly idiots on their knees in prayer .....


Paisley
Theist
Paisley's picture
Posts: 1933
Joined: 2008-03-13
User is offlineOffline
destinyawaits wrote:I'm new

destinyawaits wrote:
I'm new here and it's a bit difficult to know where to start. So I've started a new thread.

Welcome.

destinyawaits wrote:
Would someone please explain how the belief that we are finite physical beings is rational and how the belief that we are not is irrational.

You touch upon a point that I raised in my very first thread. Atheistic materialism is ultimately absurd because it views life as being ultimately absurd.

By the way, this forum frowns upon stating the obvious.

destinyawaits wrote:
I don't belong to any church or group, I had a Godless upbringing,  but I do believe there is more to life than a brief physical existence.

Agreed. To believe that life is ultimately without meaning and purpose is...well...absurd!

destinyawaits wrote:
Why should  "freethinking" be restricted to the belief that we are physical beings with a  restricted lifespan. This is a belief, a dogma like any other. It's a restriction on the imagination and very narrow filter for experience.

Agreed. The term "freethinking atheist" is actually an oxymoron. The atheist is not a free thinker, but a slave to the dogma of materialism.

"Scientists animated by the purpose of proving they are purposeless constitute an interesting subject for study." - Alfred North Whitehead


AmericanIdle
Posts: 414
Joined: 2007-03-16
User is offlineOffline
paisley wrote:Quote:Agreed.

paisley wrote:

Quote:

Agreed. The term "freethinking atheist" is actually an oxymoron. The atheist is not a free thinker, but a slave to the dogma of materialism.

My "Stupid Meter" just exploded.

When you show me anything that's even subtly compelling regarding the supernatural, we'll initiate a conversation about what that means.  Until then I'll stay grounded in the physical world as is be-fitting any rational adult. 

"Magic is religion we don't believe in.  Relgion is magic we do believe in".

Hocart

There isn't the slightest difference between them no matter how many books are written to support the distinction.  

"In a time of universal deceit, telling the truth becomes a revolutionary act."
George Orwell


Kevin R Brown
Superfan
Kevin R Brown's picture
Posts: 3142
Joined: 2007-06-24
User is offlineOffline
Paisley wrote:destinyawaits

Paisley wrote:

destinyawaits wrote:
I'm new here and it's a bit difficult to know where to start. So I've started a new thread.

Welcome.

destinyawaits wrote:
Would someone please explain how the belief that we are finite physical beings is rational and how the belief that we are not is irrational.

You touch upon a point that I raised in my very first thread. Atheistic materialism is ultimately absurd because it views life as being ultimately absurd.

By the way, this forum frowns upon stating the obvious.

destinyawaits wrote:
I don't belong to any church or group, I had a Godless upbringing,  but I do believe there is more to life than a brief physical existence.

Agreed. To believe that life is ultimately without meaning and purpose is...well...absurd!

destinyawaits wrote:
Why should  "freethinking" be restricted to the belief that we are physical beings with a  restricted lifespan. This is a belief, a dogma like any other. It's a restriction on the imagination and very narrow filter for experience.

Agreed. The term "freethinking atheist" is actually an oxymoron. The atheist is not a free thinker, but a slave to the dogma of materialism.

Paisley, please provide evidence for the claim that the Earth was made in six literal days 6000 years ago.

Quote:
"Natasha has just come up to the window from the courtyard and opened it wider so that the air may enter more freely into my room. I can see the bright green strip of grass beneath the wall, and the clear blue sky above the wall, and sunlight everywhere. Life is beautiful. Let the future generations cleanse it of all evil, oppression and violence, and enjoy it to the full."

- Leon Trotsky, Last Will & Testament
February 27, 1940


I AM GOD AS YOU
Superfan
Posts: 4793
Joined: 2007-09-29
User is offlineOffline
Hey P, you wrote. "Atheistic

Hey P, you wrote. "Atheistic materialism is ultimately absurd because it views life as being ultimately absurd."

   FUCK YOU Paisley, GOD DAMN IT !   SATAN YOU ARE !     I love Jesus ! Me GOD !

 [Edit addition 6/13]   

Pantheism must also eventually die , I ain't going for it, I AM in a hurry. I envision a future where Atheism will also have no philosophical use, as the very word "god" (the force) will be universialy understood as 100% atheist ! All is ONE.

The science questions of QM and consciousness ( C ) will have no need for "theology" nor  "isms" .....

  The placement of C regarding E/M regarding the "force" will be better understood.

  I cannot imagine C pre-existing E/M , but it is not of any major philosophical problem for me. All is ONE , connected, regardless of the ordering arrangement of the parts  F  C  E/M  etc ......

I ask the pantheists, and panentheists not to fuel the dogma of division as is the God of Abe doctrine.

Pantheism is C science philosophy. Thanks, igod as YOU. We are the "Force"!  

  


Paisley
Theist
Paisley's picture
Posts: 1933
Joined: 2008-03-13
User is offlineOffline
Kevin R Brown wrote:Paisley,

Kevin R Brown wrote:
Paisley, please provide evidence for the claim that the Earth was made in six literal days 6000 years ago.

You write this statement seemingly with the idea "I gotcha!" Sorry to disappoint, but my conception of God is slightly more sophisticated than yours. Apparently, your atheism is only limited to "Mickey Mouse" ideas of God (e.g. the "Old Man in the Sky," the "Flying Spaghetti Monster," and the "Invisible Pink Unicorn" ). This probably explains why you label yourself an atheist. If I had such an immature idea of God, I too would identify myself as an atheist. I suggest that grow up and become theologically more literate and then come back and see me.

"Scientists animated by the purpose of proving they are purposeless constitute an interesting subject for study." - Alfred North Whitehead


Paisley
Theist
Paisley's picture
Posts: 1933
Joined: 2008-03-13
User is offlineOffline
AmericanIdle wrote:My

AmericanIdle wrote:
My "Stupid Meter" just exploded.

You have my sympathy and condolences. I feel your pain. This is what happens when you attach the electrodes of your "stupid meter" to the temporal lobes of your brain.

AmericanIdle wrote:
When you show me anything that's even subtly compelling regarding the supernatural, we'll initiate a conversation about what that means.  Until then I'll stay grounded in the physical world as is be-fitting any rational adult.

Please provide me with proof that the physical is actually fundamental. Until then, atheistic materialism is just a metaphysical belief (and one which the prevailing scientific evidence does not support).

"Scientists animated by the purpose of proving they are purposeless constitute an interesting subject for study." - Alfred North Whitehead


Paisley
Theist
Paisley's picture
Posts: 1933
Joined: 2008-03-13
User is offlineOffline
I AM GOD AS YOU wrote:Hey P,

I AM GOD AS YOU wrote:
Hey P, you wrote. "Atheistic materialism is ultimately absurd because it views life as being ultimately absurd."

   FUCK YOU Paisley, GOD DAMN IT !   SATAN YOU ARE !     I love Jesus ! Me GOD !  Pantheism must also die , I ain't going for it, I AM in a hurry, 100% atheist !

Seriously, what's the problem here? Drugs or is it something else?

"Scientists animated by the purpose of proving they are purposeless constitute an interesting subject for study." - Alfred North Whitehead


Eloise
TheistBronze Member
Eloise's picture
Posts: 1808
Joined: 2007-05-26
User is offlineOffline
AmericanIdle wrote:When you

Paisley wrote:

AmericanIdle wrote:
When you show me anything that's even subtly compelling regarding the supernatural, we'll initiate a conversation about what that means.  Until then I'll stay grounded in the physical world as is be-fitting any rational adult.

Please provide me with proof that the physical is actually fundamental. Until then, atheistic materialism is just a metaphysical belief (and one which the prevailing scientific evidence does not support).

 

This point of contention just is immaterial, Paisley, no pun intended.

Materialism is not the strict metaphysical position that the physical is fundamental, though I will grant you it is interpreted that way by many materialists themselves, in essence what is also established in contemporary materialism is that what appears to be physical and what appears to be fundamental are not distinct 'things' of themselves.

It might be said that fundamental interactions are a more accurately detailed description of the physical, but in this it is clearly understood that both are descriptions of the same phenomenon, one description is given by precise mathematical instrumentation, the other by the phenomenal senses of the living being, these are distinctive abstractions, yes, but distinct things of themselves, No.

As I mentioned before, I grant you this is not always representative of individual materialist arguments, however, might you concede, in turn, that it at least represents a metaphysical position of materialism?

Theist badge qualifier : Gnostic/Philosophical Panentheist

www.mathematicianspictures.com


Paisley
Theist
Paisley's picture
Posts: 1933
Joined: 2008-03-13
User is offlineOffline
Eloise wrote:Paisley

Eloise wrote:
Paisley wrote:

AmericanIdle wrote:
When you show me anything that's even subtly compelling regarding the supernatural, we'll initiate a conversation about what that means.  Until then I'll stay grounded in the physical world as is be-fitting any rational adult.

Please provide me with proof that the physical is actually fundamental. Until then, atheistic materialism is just a metaphysical belief (and one which the prevailing scientific evidence does not support).

 

This point of contention just is immaterial, Paisley, no pun intended.

No, the point is definitely relevant because it highlights the basic difference between a materialistic worldview and a spiritual one.

Eloise wrote:
Materialism is not the strict metaphysical position that the physical is fundamental, though I will grant you it is interpreted that way by many materialists themselves

Yes, it is. Materialism is the view that physical matter is the only fundamental reality.

Quote:
materialism 1 a: a theory that physical matter is the only or fundamental reality and that all being and processes and phenomena can be explained as manifestations or results of matter

(source: Merriam-Webster Online Dictionary)

http://www.merriam-webster.com/dictionary/materialism 

Eloise wrote:
In essence what is also established in contemporary materialism is that what appears to be physical and what appears to be fundamental are not distinct 'things' of themselves.

This is the whole point. Appearances can be deceiving. Materialism is based on the belief that permanent "things" (a.k.a. as particles) constitute fundamental reality and that all change can be explained simply in terms of a reconfiguration of these particles. This is the primary illusion - that the world actually consists of permanent objects and exists independently of conscious-awareness. 

Eloise wrote:
It might be said that fundamental interactions are a more accurately detailed description of the physical, but in this it is clearly understood that both are descriptions of the same phenomenon, one description is given by precise mathematical instrumentation, the other by the phenomenal senses of the living being, these are distinctive abstractions, yes, but distinct things of themselves, No.

Materialism is based on substantialism, not processism. What you are espousing is processism and it is not compatible with materialism.

Eloise wrote:
It As I mentioned before, I grant you this is not always representative of individual materialist arguments, however, might you concede, in turn, that it at least represents a metaphysical position of materialism?

I will concede that materialists are vainly attempting to redefine the term so that it is compatible with the theory of relativity and quantum mechanics. But in so doing, they are rendering the term meaningless.

Probability waves are not physical things; they're mathematical absractions. And mathematical abstractions do not exist independent of a mind that abstracts. So why call yourself a materialist (a.k.a. physicalist) when you have dispensed with the physical? This is misleading, if not downright deceitful. To argue that probability waves and mathematical abstractions constitute fundamental reality is an argument for immaterialism, not materialism.

Quote:
immaterialism : a philosophical theory that material things have no reality except as mental perceptions

(source: Merriam-Webster Online Dictionary)

http://www.merriam-webster.com/dictionary/immaterialism 

"Scientists animated by the purpose of proving they are purposeless constitute an interesting subject for study." - Alfred North Whitehead


nigelTheBold
atheist
nigelTheBold's picture
Posts: 1868
Joined: 2008-01-25
User is offlineOffline
Paisley wrote:I will concede

Paisley wrote:

I will concede that materialists are vainly attempting to redefine the term so that it is compatible with the theory of relativity and quantum mechanics. But in so doing, they are rendering the term meaningless.

Probability waves are not physical things; they're mathematical absractions. And mathematical abstractions do not exist independent of a mind that abstracts. So why call yourself a materialist (a.k.a. physicalist) when you have dispensed with the physical? This is misleading, if not downright deceitful. To argue that probability waves and mathematical abstractions constitute fundamental reality is an argument for immaterialism, not materialism.

Here is where you and scientists diverge, my friend. Scientists accept mathematical abstractions as a model describing observation. The QM model, mathematics and all, is a model of what we observe, not an explanation of reality. The explanation of QM is still not known. This is the key: we still don't know. We have half-a-dozen potential explanations, most of which are fundamentally causal. Some are not, of course. There are some who believe that mathematics is the foundation of the universe, rather than just a logical sequence of relationships.

You seem to be claiming that since we don't know, it's God. This is fundamentally no different than the argument used by Christians, Muslims, and Preslerians the world over. It is a "God of the Gaps." The common state of knowledge in physics simply doesn't support your view of QM, and so it isn't even the general state of knowledge -- it's your ignorance that is the problem.

You are claiming knowledge in one of the hottest (and most hotly-debated) areas of physics right now: the explanation of QM. As you don't even seem to be trained as a physicist, let alone currently investigating QM, I suspect your "knowledge" is based more on the study of pop philosophy than actual physics. The way your argument goes between bad physics and philosophy, I suspect you majored (or are majoring) in philosophy.

In any case, your logic only works if you presuppose the existence of God. This is begging the question. Your arguments start well, then you jump to the conclusion that "God did it," without anything but the presupposition that God did it.

This is a poor argument indeed.

"Yes, I seriously believe that consciousness is a product of a natural process. I find that the neuroscientists, psychologists, and philosophers who proceed from that premise are the ones who are actually making useful contributions to our understanding of the mind." - PZ Myers


Kevin R Brown
Superfan
Kevin R Brown's picture
Posts: 3142
Joined: 2007-06-24
User is offlineOffline
Paisley wrote:Kevin R Brown

Paisley wrote:

Kevin R Brown wrote:
Paisley, please provide evidence for the claim that the Earth was made in six literal days 6000 years ago.

You write this statement seemingly with the idea "I gotcha!" Sorry to disappoint, but my conception of God is slightly more sophisticated than yours. Apparently, your atheism is only limited to "Mickey Mouse" ideas of God (e.g. the "Old Man in the Sky," the "Flying Spaghetti Monster," and the "Invisible Pink Unicorn&quotEye-wink. This probably explains why you label yourself an atheist. If I had such an immature idea of God, I too would identify myself as an atheist. I suggest that grow up and become theologically more literate and then come back and see me.

That was not evidence that the world was created in 6 literal days 6,000 years ago.

 

('Your' concept? How marvelous. So you've taken the Abrahamic God trope and simply romanced it into your world as best you could by put blinders on that block-out all the unreasonable aspects of Creationism.

How cute.

How arbitrary).

Quote:
"Natasha has just come up to the window from the courtyard and opened it wider so that the air may enter more freely into my room. I can see the bright green strip of grass beneath the wall, and the clear blue sky above the wall, and sunlight everywhere. Life is beautiful. Let the future generations cleanse it of all evil, oppression and violence, and enjoy it to the full."

- Leon Trotsky, Last Will & Testament
February 27, 1940


Paisley
Theist
Paisley's picture
Posts: 1933
Joined: 2008-03-13
User is offlineOffline
nigelTheBold wrote:Paisley

nigelTheBold wrote:
Paisley wrote:
I will concede that materialists are vainly attempting to redefine the term so that it is compatible with the theory of relativity and quantum mechanics. But in so doing, they are rendering the term meaningless.

Probability waves are not physical things; they're mathematical absractions. And mathematical abstractions do not exist independent of a mind that abstracts. So why call yourself a materialist (a.k.a. physicalist) when you have dispensed with the physical? This is misleading, if not downright deceitful. To argue that probability waves and mathematical abstractions constitute fundamental reality is an argument for immaterialism, not materialism.

Here is where you and scientists diverge, my friend. Scientists accept mathematical abstractions as a model describing observation. The QM model, mathematics and all, is a model of what we observe, not an explanation of reality. The explanation of QM is still not known. This is the key: we still don't know.

This is where you continue to display a basic misunderstanding of QM. You are operating under the false assumption that  quantum theory describes nature as being probabilistic simply because this represents a current lack of scientific knowledge. This simply is not true. Quantum theory teaches that nature is fundamentally indeterminate beause that is simply the way it is! If quantum theory is true, then nature is fundamentally indeterminate. End of story.

Below is the source that supports my assertion. This is the second time that I have provided you with this source. I suggest that read it and reread it (especially the underline portion) until you get it. Please stop misrepresenting the standard interpretation of QM. If you continue to do this, then I will just assume you are hell-bent on remaining willfully ignorant.

Quote:
The Copenhagen interpretation, due largely to the Danish theoretical physicist Niels Bohr, is the interpretation of quantum mechanics most widely accepted amongst physicists. According to it, the probabilistic nature of quantum mechanics predictions cannot be explained in terms of some other deterministic theory, and does not simply reflect our limited knowledge. Quantum mechanics provides probabilistic results because the physical universe is itself probabilistic rather than deterministic.

source: Wikipedia "Quantum mechanics"

http://en.wikipedia.org/wiki/Quantum_mechanics#Philosophical_consequences

nigelTheBold wrote:
We have half-a-dozen potential explanations, most of which are fundamentally causal. Some are not, of course.

I am quite aware that there are several interpretations of QM. Each qualifies as a metaphysical explanation.

By the way, the Cophenhagen interpretation is not really an interpretation. It basically states "just do the math and forget about the implications."

nigelTheBold wrote:
There are some who believe that mathematics is the foundation of the universe, rather than just a logical sequence of relationships.

Yeah, so? What's your point? That physicists and mathematicians are supporting my belief in universal mind (a.k.a. God)?

nigelTheBold wrote:
You seem to be claiming that since we don't know, it's God. This is fundamentally no different than the argument used by Christians, Muslims, and Preslerians the world over. It is a "God of the Gaps." The common state of knowledge in physics simply doesn't support your view of QM, and so it isn't even the general state of knowledge -- it's your ignorance that is the problem.

The Copenhagen interpretation states that nature is fundamentally indeterminate. This means that physical events are occurring everywhere and everywhen without physical causes. I say this evidence suggests that conscious will or mind is fundamental. What is your explanation? Some "materialism of the gaps" explanation? See...it works both ways.

Just curious, what is a "Preslerian?" I'm not familiar with this term.

 

nigelTheBold wrote:
You are claiming knowledge in one of the hottest (and most hotly-debated) areas of physics right now: the explanation of QM. As you don't even seem to be trained as a physicist, let alone currently investigating QM, I suspect your "knowledge" is based more on the study of pop philosophy than actual physics. The way your argument goes between bad physics and philosophy, I suspect you majored (or are majoring) in philosophy.

 

Quantum indeterminacy is a basic tenet of quantum theory. Ad hominem attacks will not change this fact.

nigelTheBold wrote:
In any case, your logic only works if you presuppose the existence of God. This is begging the question. Your arguments start well, then you jump to the conclusion that "God did it," without anything but the presupposition that God did it.

Yeah, what question is it begging?

"Scientists animated by the purpose of proving they are purposeless constitute an interesting subject for study." - Alfred North Whitehead


Eloise
TheistBronze Member
Eloise's picture
Posts: 1808
Joined: 2007-05-26
User is offlineOffline
Paisley wrote:I will concede

Paisley wrote:

I will concede that materialists are vainly attempting to redefine the term so that it is compatible with the theory of relativity and quantum mechanics. But in so doing, they are rendering the term meaningless. Probability waves are not physical things; they're mathematical absractions. And mathematical abstractions do not exist independent of a mind that abstracts. So why call yourself a materialist (a.k.a. physicalist) when you have dispensed with the physical?

No, the point that you're missing is that it is not a redefinition of materialism, but a definition of matter that sits at the heart of this and so it is the implication of materialism which is changed, not the definition. Matter as a process of interactions is still matter, and matter as the physicalist defines it remains an entirely valid proposition; the distinction is that it is formed from observations made at another level.  The process description and the physical description are abstractions, what they describe is what exists, and they do not describe distinct things. Therefore, materialism, the view that this abstraction 'physical' applies to all fundamental things, that all things are matter, remains perfectly valid, it is not dispensed with at all.

This brings us to the relevant contention which is between neutral monism and physical monism. Neutral monism takes no sides between the abstractions, between the technical observation (experimentation) and the personal observation (sensory reality), neutral monism gives both abstractions equal weight as descriptors of matter whereas physical monism sees the technical observation as referring objectively to the personal observation so they're weighted unevenly according to their type.

Paisley wrote:

This is misleading, if not downright deceitful. To argue that probability waves and mathematical abstractions constitute fundamental reality is an argument for immaterialism, not materialism.

It's not immaterialism, and it's not an argument that mathematical abstractions are fundamental reality. One can argue that fundamental reality correlates so precisely well to mathematics that what is fundamentally real may be mathematical in nature, but this is not to say that our mathematical abstractions are the reality, only that mathematical language is our best tool for preserving the state of the observed reality into abstraction.

I continue to grant you that the concept of materialism can be very misleading, but the position of materialism itself is not undermined by the error of it's adherents. Materialism is sound enough, but it desperately needs wider discussion.

Theist badge qualifier : Gnostic/Philosophical Panentheist

www.mathematicianspictures.com


Paisley
Theist
Paisley's picture
Posts: 1933
Joined: 2008-03-13
User is offlineOffline
Kevin R Brown wrote:That was

Kevin R Brown wrote:
That was not evidence that the world was created in 6 literal days 6,000 years ago.

I will assume that you are familiar with the term "straw-man argument." Enough said.

Kevin R Brown wrote:
('Your' concept? How marvelous. So you've taken the Abrahamic God trope and simply romanced it into your world as best you could by put blinders on that block-out all the unreasonable aspects of Creationism.

How cute.

How arbitrary).

The choice was between mindless matter or elemental mind. I chose the latter. I like my position better than yours.

Panentheists like myself generally subscribe to emanationism rather than creationism. 

"Scientists animated by the purpose of proving they are purposeless constitute an interesting subject for study." - Alfred North Whitehead


Kevin R Brown
Superfan
Kevin R Brown's picture
Posts: 3142
Joined: 2007-06-24
User is offlineOffline
Hey Paisley, just what the

Hey Paisley, just what the Hell is your concept of God anyway, given that it's apparently not the standard creationist trope?

 

This oughta be good.

Quote:
"Natasha has just come up to the window from the courtyard and opened it wider so that the air may enter more freely into my room. I can see the bright green strip of grass beneath the wall, and the clear blue sky above the wall, and sunlight everywhere. Life is beautiful. Let the future generations cleanse it of all evil, oppression and violence, and enjoy it to the full."

- Leon Trotsky, Last Will & Testament
February 27, 1940


Paisley
Theist
Paisley's picture
Posts: 1933
Joined: 2008-03-13
User is offlineOffline
Kevin R Brown wrote:Hey

Kevin R Brown wrote:

Hey Paisley, just what the Hell is your concept of God anyway, given that it's apparently not the standard creationist trope?

 

This oughta be good.

See post #68.

"Scientists animated by the purpose of proving they are purposeless constitute an interesting subject for study." - Alfred North Whitehead


Kevin R Brown
Superfan
Kevin R Brown's picture
Posts: 3142
Joined: 2007-06-24
User is offlineOffline
'Elemental mind'? How

'Elemental mind'? How ambiguous.

You think that fire and electricity can think and shit, then?

EDIT:

Quote:

I will assume that you are familiar with the term "straw-man argument." Enough said.

I'll assume you're familiar with the term 'trope'? As in, 'You stole the 'God' trope from creationism in the first place'?

Because that blows your claim that I set-up a strawman out of the water.

Quote:
"Natasha has just come up to the window from the courtyard and opened it wider so that the air may enter more freely into my room. I can see the bright green strip of grass beneath the wall, and the clear blue sky above the wall, and sunlight everywhere. Life is beautiful. Let the future generations cleanse it of all evil, oppression and violence, and enjoy it to the full."

- Leon Trotsky, Last Will & Testament
February 27, 1940


nigelTheBold
atheist
nigelTheBold's picture
Posts: 1868
Joined: 2008-01-25
User is offlineOffline
Paisley wrote:This is where

Paisley wrote:

This is where you continue to display a basic misunderstanding of QM. You are operating under the false assumption that  quantum theory describes nature as being probabilistic simply because this represents a current lack of scientific knowledge. This simply is not true. Quantum theory teaches that nature is fundamentally indeterminate beause that is simply the way it is! If quantum theory is true, then nature is fundamentally indeterminate. End of story.

And exactly where did you study QM?

Mine was the physics department of University of Alaska, Fairbanks. There we learned that the Copenhagen interpretation (which you quote below) simply stated that "indeterminate" meant "can't be predicted," and does not mean, "is not caused." As I discussed above, but which you so blithely ignored, randomness from a deterministic source (such as the Perl rand function) is indistinguishable from "pure" randomness, assuming the implementation is unknown. If you can't tell the difference, the outcome will be "indeterminate." And so chaotic energy fluctuations at the planck scale are sufficient to explain QM, while remaining firmly in the realm of materialism.

End of story.

But unfortunately, you missed that fact during your QM studies.

Quote:

Below is the source that supports my assertion. This is the second time that I have provided you with this source. I suggest that read it and reread it (especially the underline portion) until you get it. Please stop misrepresenting the standard interpretation of QM. If you continue to do this, then I will just assume you are hell-bent on remaining willfully ignorant.

Quote:
The Copenhagen interpretation, due largely to the Danish theoretical physicist Niels Bohr, is the interpretation of quantum mechanics most widely accepted amongst physicists. According to it, the probabilistic nature of quantum mechanics predictions cannot be explained in terms of some other deterministic theory, and does not simply reflect our limited knowledge. Quantum mechanics provides probabilistic results because the physical universe is itself probabilistic rather than deterministic.

source: Wikipedia "Quantum mechanics"

http://en.wikipedia.org/wiki/Quantum_mechanics#Philosophical_consequences

Wilfully ignorant? I at least have studied the subject. I don't believe you have at all. Otherwise, you would not try to interpret the Copenhagen Interpretation as supporting "uncaused" events.

Chaos is probabilistic. Just so you know. It is also deterministic, in the strictest sense of the word, but that doesn't stop it from being probabilistic.

Quote:

nigelTheBold wrote:
We have half-a-dozen potential explanations, most of which are fundamentally causal. Some are not, of course.

I am quite aware that there are several interpretations of QM. Each qualifies as a metaphysical explanation.

Noooooo, each qualifies as an hypothesis. Have you even bothered to study how science works? Or are you just winging it here?

Quote:

By the way, the Cophenhagen interpretation is not really an interpretation. It basically states "just do the math and forget about the implications."

Yes. Basically it says, "We have no clue what's going on, but this is how it behaves." The math is a model of behavior which indicates that QM follows a probability distribution. That's it. The "explanation" wasn't covered, because there was no model yet to describe it. Now we have a half-dozen (or more) likely explanations, which are called "hypothesis." We don't know which is correct, but in all likelihood, QM is causal in nature.

Do your fucking research.

Quote:

nigelTheBold wrote:
There are some who believe that mathematics is the foundation of the universe, rather than just a logical sequence of relationships.

Yeah, so? What's your point? That physicists and mathematicians are supporting my belief in universal mind (a.k.a. God)?

No, that some people do believe mathematics is fundamental to the universe. And I have no clue where the "universal mind" bullshit comes from, unless you believe God is mathematics.

Quote:

nigelTheBold wrote:
You seem to be claiming that since we don't know, it's God. This is fundamentally no different than the argument used by Christians, Muslims, and Preslerians the world over. It is a "God of the Gaps." The common state of knowledge in physics simply doesn't support your view of QM, and so it isn't even the general state of knowledge -- it's your ignorance that is the problem.

The Copenhagen interpretation states that nature is fundamentally indeterminate. This means that physical events are occurring everywhere and everywhen without physical causes. I say this evidence suggests that conscious will or mind is fundamental. What is your explanation? Some "materialism of the gaps" explanation? See...it works both ways.

*sigh*

No. It. Does. Not.

Indeterminacy means it's not predictable. That is not the same as "without cause." If you'd bother to actually study QM instead of read popular synopsis, you might understand. It'd be damned fine if you would, so I wouldn't have to keep repeating myself, and you could stop ignoring those rebuttals.

Quote:

Just curious, what is a "Preslerian?" I'm not familiar with this term.

I'd hope not. I just made it up as a shorthand for the Church of Elvis. (Which is a real church.) I just didn't want things to get too serious.

Quote:

nigelTheBold wrote:
You are claiming knowledge in one of the hottest (and most hotly-debated) areas of physics right now: the explanation of QM. As you don't even seem to be trained as a physicist, let alone currently investigating QM, I suspect your "knowledge" is based more on the study of pop philosophy than actual physics. The way your argument goes between bad physics and philosophy, I suspect you majored (or are majoring) in philosophy.

Quantum indeterminacy is a basic tenet of quantum theory. Ad hominem attacks will not change this fact.

I'm glad you see that being called a philosopher is a bad thing. That at least means we might have some common ground.

However, you still haven't supported your "knowledge." Your base claims are completely incorrect, which leaves your thesis... anemic. To say the least.

"Yes, I seriously believe that consciousness is a product of a natural process. I find that the neuroscientists, psychologists, and philosophers who proceed from that premise are the ones who are actually making useful contributions to our understanding of the mind." - PZ Myers


I AM GOD AS YOU
Superfan
Posts: 4793
Joined: 2007-09-29
User is offlineOffline
Paisley wrote:I AM GOD AS

Paisley wrote:

I AM GOD AS YOU wrote:
Hey P, you wrote. "Atheistic materialism is ultimately absurd because it views life as being ultimately absurd."

   FUCK YOU Paisley, GOD DAMN IT !   SATAN YOU ARE !     I love Jesus ! Me GOD !  Pantheism must also die , I ain't going for it, I AM in a hurry, 100% atheist !

Seriously, what's the problem here? Drugs or is it something else?

In the spirit of my righteously indignant atheistic Jesus, and in modern style of linguists, as fowl mouth George Carlin, I strongly disagree with you.  And as I have said a few times, thank you P sincerely for your posts that we can bounce communication, so all can learn of what we are. We are Fucking ignorant is our first revelation to wisdom. Please feel free to cuss at me. Can you cuss, or is that against your pantheism world view?

Drunk or not, I love you more, Fucker ....   

My point is some things aren't worth actually debating and need to be attacked, to heal. Like all dogma ..... Fuck Dogma, god damn it !  ..... Get (the fuck out of my way) SATAN, Jesus said to Peter ....  -------------------------

Yeah Eloise , teacher of the ONENESS, as you say, "a metaphysical position of materialism" ....   Yep, and when we understand it will no longer be metaphysical. We are simply ignorant, As Nigel the Bold said, "We don't know" .....

    Why make shit up P and spout it as truth? Keep the awe, stop the dogma ....

 


zarathustra
atheist
zarathustra's picture
Posts: 1521
Joined: 2006-11-16
User is offlineOffline
Paisley and the Perpetual Ambiguity

Kevin R Brown wrote:

Hey Paisley, just what the Hell is your concept of God anyway, given that it's apparently not the standard creationist trope?

It's pointless to ask paisley to define his concept of god (or make any definitive statement for that matter).  He changes his definitions more often than the weather: 

I have stated before (either here or in another thread) that I subscribe to pantheism/panentheism. I allow myself the luxury to change theological views as I see appropriate. So, this is not really an issue.


 

There are no theists on operating tables.

πππ†
π†††


jcgadfly
Superfan
Posts: 6791
Joined: 2006-07-18
User is offlineOffline
zarathustra wrote:Kevin R

zarathustra wrote:

Kevin R Brown wrote:

Hey Paisley, just what the Hell is your concept of God anyway, given that it's apparently not the standard creationist trope?

It's pointless to ask paisley to define his concept of god (or make any definitive statement for that matter).  He changes his definitions more often than the weather: 

I have stated before (either here or in another thread) that I subscribe to pantheism/panentheism. I allow myself the luxury to change theological views as I see appropriate. So, this is not really an issue.


 

Which is why he's basically an atheist - his "god" is so watered down and useless to be of any real significance in his life.

Dawkins has him pegged - no matter how often he tries to get away.

"I do this real moron thing, and it's called thinking. And apparently I'm not a very good American because I like to form my own opinions."
— George Carlin


draggycat
draggycat's picture
Posts: 38
Joined: 2007-10-08
User is offlineOffline
 Paisley wrote: You touch

 

Paisley wrote:

 

You touch upon a point that I raised in my very first thread. Atheistic materialism is ultimately absurd because it views life as being ultimately absurd.


 

pardon me, but weren't you already called on this treatment of the word "absurd" before? blurring two different possible usages like this, your sentence could just as easily read as "Atheistic materialism is ultimately aimless because it views life as being ultimately amusing."


Paisley
Theist
Paisley's picture
Posts: 1933
Joined: 2008-03-13
User is offlineOffline
nigelTheBold wrote:Paisley

nigelTheBold wrote:
Paisley wrote:
This is where you continue to display a basic misunderstanding of QM. You are operating under the false assumption that quantum theory describes nature as being probabilistic simply because this represents a current lack of scientific knowledge. This simply is not true. Quantum theory teaches that nature is fundamentally indeterminate because that is simply the way it is! If quantum theory is true, then nature is fundamentally indeterminate. End of story.

And exactly where did you study QM?

Mine was the physics department of University of Alaska, Fairbanks. There we learned that the Copenhagen interpretation (which you quote below) simply stated that "indeterminate" meant "can't be predicted," and does not mean, "is not caused." As I discussed above, but which you so blithely ignored, randomness from a deterministic source (such as the Perl rand function) is indistinguishable from "pure" randomness, assuming the implementation is unknown. If you can't tell the difference, the outcome will be "indeterminate." And so chaotic energy fluctuations at the planck scale are sufficient to explain QM, while remaining firmly in the realm of materialism.

End of story.

But unfortunately, you missed that fact during your QM studies.

You provide me with your resume to support your arguments; I cite credible sources to support mine. That's the difference between my posts and yours.

Paul Davies (science writer and physics professor at Arizona State University) states emphatically and unequivocally that "quantum events are not determined by preceding causes." This is why it is called indeterminism. Duh!

Quote:
At the heart of quantum physics lies Heisenberg's uncertainty principle, which states that all measurable quantities (e.g. position, momentum, energy) are subject to unpredictable fluctuations in their values. This unpredictability implies that the microworld is indeterministic: to use Einstein's picturesque phraseology, God plays dice with the universe. Therefore, quantum events are not determined absolutely by preceding causes. Although the probability of a given event (e.g. the radioactive decay of an atomic nucleus) is fixed by the theory, the actual outcome of a particular quantum process is unknown and, even in principle, unknowable.

(source: pg. 61 "Mind of God: the Scientific Basis for a Rational World" by Paul Davies)

Chaos theory states that minor changes of data in the initial conditions of a dynamic system can be exponentially amplified to such an extent that as the system evolves over time it becomes impossible to predict (for all intents and practical purposes) the effect that such minor changes will have on the final state of that system (e.g. forecasting a weather system). However, chaos theory is a deterministic theory, not an indeterminate one. (I have already cited sources to support this claim.) Note that chaos theory is considered to be UNPREDICTABLE and yet DETERMNISTIC. Why? Because the unpredictability is based on practical concerns of the complexity involved in determining how all the cause and effect relationships will play out whenever minor changes are made to the data of the initial conditions of a system. However, the same cannot be said of QM. Quantum events are unpredictable precisely because they are INDETERMINATE. They (i.e. the quantum events) are believed to be inherently unpredictable (not because of complexity) but because they are "pure chance" events and without cause. This is what you're not getting!

Also, you are appealing to speculation by stating "chaotic energy fluctuations at the planck scale are sufficient to explain QM, while remaining firmly in the realm of materialism." This is not quantum theory. This is not a scientific theory that has been tested and verified. This is mere speculation! Quantum theory is not based on chaos theory.

I suspect that the connection between chaos theory and quantum mechanics may be to provide an explanation how small perturbations (random quantum fluctuations) in the microworld (i.e. quantum events) can have a major effect in the macroworld. The argument is sometimes made that quantum events are only relevant in the microworld of subatomic particles, not in the macroworld of Newtonian physics. Chaos theory can provide a basis to explain why this is not so and that the macroworld may also be indeterminate.

nigelTheBold wrote:
Paisley wrote:
Below is the source that supports my assertion. This is the second time that I have provided you with this source. I suggest that read it and reread it (especially the underline portion) until you get it. Please stop misrepresenting the standard interpretation of QM. If you continue to do this, then I will just assume you are hell-bent on remaining willfully ignorant.

Quote:
The Copenhagen interpretation, due largely to the Danish theoretical physicist Niels Bohr, is the interpretation of quantum mechanics most widely accepted amongst physicists. According to it, the probabilistic nature of quantum mechanics predictions cannot be explained in terms of some other deterministic theory, and does not simply reflect our limited knowledge. Quantum mechanics provides probabilistic results because the physical universe is itself probabilistic rather than deterministic.

source: Wikipedia "Quantum mechanics"

http://en.wikipedia.org/wiki/Quantum_mechanics#Philosophical_consequences

Wilfully ignorant? I at least have studied the subject. I don't believe you have at all. Otherwise, you would not try to interpret the Copenhagen Interpretation as supporting "uncaused" events.

Yes, willfully ignorant! Once again, you give me your resume while I provide you with a credible source. I said to read it and reread it until you get it. It says that "the probabilistic nature of quantum mechanics predictions cannot be explained in terms of some other deterministic theory, and does not simply reflect our limited knowledge." Did you get it? It cannot be explained in terms of some other DETERMINISTIC theory (such as Chaos Theory)!

Now, unless you can provide me with a credible source that refutes quantum indeterminacy, then don't bother responding to this post. Don't give me your resume, give me an authoritative source. If you can do that and stop flinging personal insults, then maybe I would be more willing to consider what you have to say.

Quote:
Chaos is probabilistic. Just so you know. It is also deterministic, in the strictest sense of the word, but that doesn't stop it from being probabilistic.

I am fully aware that probability theory has numerous applications and is used to describe processes that, for all intents and practical purposes, are considered to be random. However, probabilities in the context of QM is being used to describe a phenomenon that is believed to be a "pure chance" event. This is what you're not getting!

I will grant you that quantum theory may be falsified sometime in the future (all scientific theories are subject to being falsified). And what was previously believed to be a pure chance event may then be viewed as only having the appearance of a random event. But until quantum theory is falsified, then you do not have the luxury of saying that we do not have scientific evidence for "uncaused events." We most certainly do. This is why I can say that the prevailing scientific evidence suggests that materialism is invalid because it has been undermined by quantum mechanics.

nigelTheBold wrote:
Paisley wrote:
I am quite aware that there are several interpretations of QM. Each qualifies as a metaphysical explanation.

Noooooo, each qualifies as an hypothesis. Have you even bothered to study how science works? Or are you just winging it here?

Previously, you described theses hypotheses with an acronym - I believe the term you used was "SWAG" or something like that. Can you please define what the acronym stands for. I can't remember. Thank you.

A scientific hypothesis must be testable and falsifiable.

http://en.wikipedia.org/wiki/Hypothesis

The many-worlds interpretation (MWI) is actually an interpretation that is held by most scientists (a whopping 60%).

http://en.wikipedia.org/wiki/Many-worlds_interpretation

Do you believe that MWI is testable and falsifiable? Please explain to me how 60% of scientists believe in an infinite number of parallel universes for which there is absolutely no scientific proof? I would say this smacks of metaphysics. What do you think?

nigelTheBold wrote:
Paisley wrote:
By the way, the Cophenhagen interpretation is not really an interpretation. It basically states "just do the math and forget about the implications."

Yes. Basically it says, "We have no clue what's going on, but this is how it behaves." The math is a model of behavior which indicates that QM follows a probability distribution. That's it. The "explanation" wasn't covered, because there was no model yet to describe it. Now we have a half-dozen (or more) likely explanations, which are called "hypothesis." We don't know which is correct, but in all likelihood, QM is causal in nature.

I did do my research. Thank you very much. And this is why I can say there are no "explanations" for "uncaused events" (not unless you consider metaphysical ones).

Incidentally, quantum indeterminacy is not the only aspect of QM that undermines materialism. There are other aspects - namely, quantum superposition (that an electron can apparently be in all places at the same time prior to collaspe of the wave function), quantum entanglement (what Einstein called "spooky action at a distance" ), and nonlocality (information is exchanged superluminally (faster than the speed of light)....actually information is exchanged simultaneously). Take all this account and then factor Bell's theorem into the mix, which states that there is no deterministic theory (hidden local variables) that can reproduce all the predictions made by quantum theory, and you will basically sound the death knell of materialism. May it forever rest in peace.

Quote:
Bell's theorem is a theory that shows that the predictions of quantum mechanics (QM) are not intuitive, and touches upon fundamental philosophical issues that relate to modern physics. It is the most famous legacy of the late physicist John S. Bell. Bell's theorem states:

"No physical theory of local hidden variables can ever reproduce all the predictions of quantum mechanics."

source: Wikipedia "Bell's theorem"

http://en.wikipedia.org/wiki/Bell%27s_inequality

nigelTheBold wrote:
Paisley wrote:
nigelTheBold wrote:
There are some who believe that mathematics is the foundation of the universe, rather than just a logical sequence of relationships.

Yeah, so? What's your point? That physicists and mathematicians are supporting my belief in universal mind (a.k.a. God)?

No, that some people do believe mathematics is fundamental to the universe. And I have no clue where the "universal mind" bullshit comes from, unless you believe God is mathematics.

It's called platonic forms (e.g. mathematical abstractions) and presupposes a universal mind (the idealism of Plato). If scientists make the argument that mathematics forms the structure of ultimate reality (rather than merely describing certain aspects of it), then they are making a tacit argument for some kind of pantheistic idealism. By the way, I have read that physicists and mathematicians have a much higher rate of theistic belief than their colleagues in the other natural sciences (e.g. chemistry and biology). I suspect this due to the argument I just made.

Quote:
Platonic idealism is the theory that the substantive reality around us is only a reflection of a higher truth. That truth, Plato argued, is the abstraction. He believed that ideas were more real than things. He developed a vision of two worlds: a world of unchanging ideas and a world of changing physical objects.

Platonism is considered to be, in mathematics departments the world over, the predominant philosophy of mathematics, especially regarding the foundations of mathematics.

source: Wikipedia "Platonic Idealism"

http://en.wikipedia.org/wiki/Platonic_idealism

nigelTheBold wrote:
I'm glad you see that being called a philosopher is a bad thing. That at least means we might have some common ground.

Your contempt for philosophical reflection probably explains why you have failed to acquire a basic understanding of the term "indeterminism" and what it implies.

"Scientists animated by the purpose of proving they are purposeless constitute an interesting subject for study." - Alfred North Whitehead


BobSpence
High Level DonorRational VIP!ScientistWebsite Admin
BobSpence's picture
Posts: 5939
Joined: 2006-02-14
User is offlineOffline
Quote:Therefore, quantum

Quote:

Therefore, quantum events are not determined absolutely by preceding causes. Although the probability of a given event (e.g. the radioactive decay of an atomic nucleus) is fixed by the theory, the actual outcome of a particular quantum process is unknown and, even in principle, unknowable.

This is NOT stating that the outcome is uncaused - just that the the PARTICULAR outcome of a particular quantum event, within the space of possible outcomes, is not UNIQUELY determined by preceding causes. Instead the probability of any specified outcome is precisely defined by a wave function, which for many quantum events we have accurate mathematical expression for.

This does not mean that the event is uncaused. Paisley has yet to provide a reference supporting his assertion that Quantum indetermacy means strictly "uncaused". If this is the closest you can get  to a respected scientist supporting your proposition, then you should be honest enough to admit you are out on a limb and have made an unwarranted conclusion, and need to re-think. I definitely respect Paul Davies, and find his recent thoughts on the origin of the universe, the 'fine-tuning' argument. etc, very interesting and thought-provoking.

For example, on the 'Multiple universe' hypothesis, which you seem so dismissive of:

Paul Davies wrote:
Many cosmologists believe that the universe we observe is an infinitesimal component of a "multiverse," or ensemble of universes, with differing properties and laws. I am developing some new ideas for testing this hypothesis.

and also somewhat relevant to this discussion:

Quote:
The status of the laws of physics - what are they and where do they come from? - is an old philosophical problem that has received a new twist with modern scientific ideas like string theory and the multiverse. I am developing a new slant on this problem by investigating whether the laws of physics might be emergent, rather than absolute, universal and Platonic. (See my paper for more about this topic). I have suggested a link between the way the laws of physics operate and the large scale properties of the universe. This opens the way to the possibility of hitherto unknown "higher level" emergent laws, such as biological principles of organization, that are consistent with, but not reducible to, the traditional laws of physics. In connection with this project, I am trying to sharpen the concept of "downward causation", where wholes have causal efficacy over parts. I am also seeking to eliminate the age-old dualism between absolute unchanging laws of physics and time-dependant contingent states of the world, and develop a notion where laws and states might co-evolve.

Anyway, back to indeterminacy.

The indeterminacy can readily be seen as a direct consequence of the Heisenberg blurring of the position/momentum of particles, introducing an irreducible uncertainty or 'fuzziness' into the outcome of any interaction. So the issue is NOT uncaused events, it is why is there this ultimate limit on the precision with which the ultimate state of the Universe can be specified? At least the purity of its randomness suggests it is a natural, 'mechanistic', effect....

Favorite oxymorons: Gospel Truth, Rational Supernaturalist, Business Ethics, Christian Morality

"Theology is now little more than a branch of human ignorance. Indeed, it is ignorance with wings." - Sam Harris

The path to Truth lies via careful study of reality, not the dreams of our fallible minds - me

From the sublime to the ridiculous: Science -> Philosophy -> Theology


nigelTheBold
atheist
nigelTheBold's picture
Posts: 1868
Joined: 2008-01-25
User is offlineOffline
Paisley wrote:I suspect that

Paisley wrote:

I suspect that the connection between chaos theory and quantum mechanics may be to provide an explanation how small perturbations (random quantum fluctuations) in the microworld (i.e. quantum events) can have a major effect in the macroworld. The argument is sometimes made that quantum events are only relevant in the microworld of subatomic particles, not in the macroworld of Newtonian physics. Chaos theory can provide a basis to explain why this is not so and that the macroworld may also be indeterminate.

On that, we definitely agree. (Except, I suspect, for our definition of "indeterminate." )

Quote:

Now, unless you can provide me with a credible source that refutes quantum indeterminacy, then don't bother responding to this post.

See, here's the problem: you are mistaking philosophical indeterminacy with quantum indeterminacy. They are not the same.

Quantum indeterminacy means simply that you cannot measure a unique state of a quantum system. You can measure a single property, but not the full, unique state. That is, it is "unknowable." If you read the Wiki article on quantum indeterminacy, you will see there's not one mention of "uncaused" events.

Scientists are not in agreement on this subject. For instance, the quantum decoherence crowd believes that every possible superposition is realized, resulting in the "multi-universe" concept. This essentially means that each instance of the superposition isn't random, but inevitable.

Another example:  Gerard 't Hooft proposes a model which is strictly chaotic, in which particles and their properties are composed of planck-scale artefacts.

Then there's your assertion that the Copenhagen Interpretation is non-causal:

John G. Cramer, University of Washington wrote:

While this element of the [Copenhagen Interpretation] may not satisfy the desires of some physicists for a completely predictive and deterministic theory, it must be considered as at least an adequate solution to the problem unless a better alternative can be found. Perhaps the greatest weakness of [the Copenhagen Interpretation] in this context is not that it asserts an intrinsic randomness but that it supplies no insight into the nature or origin of this randomness.

The Copenhagen Interpretation specifies only that the system is statistical in nature, not that it is uncaused. The CI specifically does not provide "insight into the nature" of the randomness.

Quote:

I am fully aware that probability theory has numerous applications and is used to describe processes that, for all intents and practical purposes, are considered to be random. However, probabilities in the context of QM is being used to describe a phenomenon that is believed to be a "pure chance" event. This is what you're not getting!

That has not even remotely been established. Not by you, not by any physicist. Not by anyone. That's what you're not getting.

Physicists studying QM can't even agree on the nature of the randomness, as I hope I have successfully established. Your constant assertions that they have is inconsistent with reality.

One of the reasons I gave you my "resume" was to provoke you into revealing you are a philosopher, most likely a philosophy professor. In my experience, they are the only ones who insist (as you do) that the "uncaused events" is the sole possible interpretation of Quantum Mechanics. Although physicists often have their own beliefs (such as the many-worlds interpretation of quantum decoherence), most will admit that they have no proof (yet), and that other possibilities (espoused by other physicists) are also likely.

I agree that uncaused events is one possible interpretation. I get that. However, it's not the only possible interpretation; nor is it the one that is most accepted. (As I pointed out, the CI does not specify whether the randomness is "caused" or "uncaused." )

Therefore, you can talk of probabilities, and of potential consequences. However, you cannot state categorically that quantum events are uncaused and remain in the realm of the current ontology.

Now, our problem may be that we are simply quibbling over the meaning of "causal" events. But I don't think so, as you seem to get that chaos and "pure chance" are distinct.

Quote:

Previously, you described theses hypotheses with an acronym - I believe the term you used was "SWAG" or something like that. Can you please define what the acronym stands for. I can't remember. Thank you.

A scientific hypothesis must be testable and falsifiable.

"Scientific Wild-Assed Guess."

Quote:

 

The many-worlds interpretation (MWI) is actually an interpretation that is held by most scientists (a whopping 60%).

http://en.wikipedia.org/wiki/Many-worlds_interpretation

Do you believe that MWI is testable and falsifiable? Please explain to me how 60% of scientists believe in an infinite number of parallel universes for which there is absolutely no scientific proof? I would say this smacks of metaphysics. What do you think?

It definitely smacks of metaphysics. I agree. It's an outcome that is predicted by quantum decoherence, but there's no way to access data on whether or not these "other worlds" even exist.

Definitely sounds like metaphysics to me.

Quote:

I did do my research. Thank you very much. And this is why I can say there are no "explanations" for "uncaused events" (not unless you consider metaphysical ones).

Incidentally, quantum indeterminacy is not the only aspect of QM that undermines materialism. There are other aspects - namely, quantum superposition (that an electron can apparently be in all places at the same time prior to collaspe of the wave function), quantum entanglement (what Einstein called "spooky action at a distance" ), and nonlocality (information is exchanged superluminally (faster than the speed of light)....actually information is exchanged simultaneously). Take all this account and then factor Bell's theorem into the mix, which states that there is no deterministic theory (hidden local variables) that can reproduce all the predictions made by quantum theory, and you will basically sound the death knell of materialism. May it forever rest in peace.

Bell's theorom rules out local variables. It doesn't rule out global variables. (Planck-scale energy variances, the same phenomena that potentially gives rise to virtual particles.)

I'll get to the others in a bit. Gotta run.

"Yes, I seriously believe that consciousness is a product of a natural process. I find that the neuroscientists, psychologists, and philosophers who proceed from that premise are the ones who are actually making useful contributions to our understanding of the mind." - PZ Myers


Eloise
TheistBronze Member
Eloise's picture
Posts: 1808
Joined: 2007-05-26
User is offlineOffline
Paisley wrote:The

 

Paisley wrote:

The many-worlds interpretation (MWI) is actually an interpretation that is held by most scientists (a whopping 60%).

http://en.wikipedia.org/wiki/Many-worlds_interpretation

Do you believe that MWI is testable and falsifiable? Please explain to me how 60% of scientists believe in an infinite number of parallel universes for which there is absolutely no scientific proof? I would say this smacks of metaphysics. What do you think?

Ok stop right there, this is ludicrous, you're arguing the collapse postulate and telling us you reject MWI on lack of scientific evidence? I take it you can prove the collapse postulate then? Falsify interference at size levels above the buckyball? There is no more evidence for collapse than there is for no collapse, Paisley, so please get your facts straight before you shoot low.

I'll have you know the relative state formulation is rigorously mathematical (identical to other formulations of QM in this sense) and thoroughly well supported by many of the best and most noteworthy advances in quantum cosmology and quantum technology in the last decade (in all other respects). To swipe a clever retort from DG there are a great many things that just cannot make sense except in the context of the theory of all quantum states being actualised. 60% of physicists (is it really that high these days? Wow!) believe it because so consistently are successful advances in the theory tracing back to it in one form or another. 

Oh and one more thing: this time you can rightly suppose that you have learned something about what I believe, because this time I am arguing my position, not someone else's.

PS: If you are going to insist that no collapse is not scientifically verified while simultaneously arguing that the collapse postulate is absolute truth, then explain how the quantum eraser experiment can restore the wavefunction post perturbation while you're at it.

Theist badge qualifier : Gnostic/Philosophical Panentheist

www.mathematicianspictures.com


I AM GOD AS YOU
Superfan
Posts: 4793
Joined: 2007-09-29
User is offlineOffline
Eloise kicks ass

Eloise kicks ass


BMcD
Posts: 777
Joined: 2006-12-20
User is offlineOffline
Paisley wrote:You touch upon

Paisley wrote:

You touch upon a point that I raised in my very first thread. Atheistic materialism is ultimately absurd because it views life as being ultimately absurd.

By the way, this forum frowns upon stating the obvious.

And we've responded to this many times, Paisley. You're using two different definitions of 'absurd' and then claiming they are the same thing.

 

A world view which holds that there is no objective 'ultimate' meaning behind existence is not necessarily a world view that makes no sense or is ridiculous. It may make no sense, and it may be ridiculous, but that it does not claim an 'ultimate' meaning does not, in and of itself, make it either.

 

What you claim is basically and functionally the same as saying: "Smurf is ultimately smurfy because it smurfs smurf as being ultimately smurfy."

"You've got to remember that these are just simple farmers. These are people of the land. The common clay of the new West. You know... morons." - The Waco Kid


nigelTheBold
atheist
nigelTheBold's picture
Posts: 1868
Joined: 2008-01-25
User is offlineOffline
BMcD wrote:"Smurf is

BMcD wrote:

"Smurf is ultimately smurfy because it smurfs smurf as being ultimately smurfy."

Truer words have never been said.

"Yes, I seriously believe that consciousness is a product of a natural process. I find that the neuroscientists, psychologists, and philosophers who proceed from that premise are the ones who are actually making useful contributions to our understanding of the mind." - PZ Myers


Paisley
Theist
Paisley's picture
Posts: 1933
Joined: 2008-03-13
User is offlineOffline
nigelTheBold wrote:Paisley

nigelTheBold wrote:
Paisley wrote:

I suspect that the connection between chaos theory and quantum mechanics may be to provide an explanation how small perturbations (random quantum fluctuations) in the microworld (i.e. quantum events) can have a major effect in the macroworld. The argument is sometimes made that quantum events are only relevant in the microworld of subatomic particles, not in the macroworld of Newtonian physics. Chaos theory can provide a basis to explain why this is not so and that the macroworld may also be indeterminate.

On that, we definitely agree. (Except, I suspect, for our definition of "indeterminate." )

Good. But please explain the difference between a theory that is...

1) unpredictable and yet deterministic (e.g. chaos theory), and one that is...

2) unpredictable and indeterministic (e.g. quantum theory)

nigelTheBold wrote:
See, here's the problem: you are mistaking philosophical indeterminacy with quantum indeterminacy. They are not the same.

Quantum indeterminacy means simply that you cannot measure a unique state of a quantum system. You can measure a single property, but not the full, unique state. That is, it is "unknowable." If you read the Wiki article on quantum indeterminacy, you will see there's not one mention of "uncaused" events.

I know that indeterminism is a philosophical belief. But this does not mean that indeterminism in the context of science means something different.

The Wikipedia article (philosophical indeterminacy) that you cited (and by the way, congratulations for citing a source) states that quantum mechanics implies indeterminacies (using the term in the philosophical sense). Also, the other article (quantum indeterminacy), which you have cited, doesn't have to say "uncaused" events. The definition of the term indeterminism or indeterminacy implies this.

In the Wikipedia article entitled "Indeterminism," the term is defined as being contradictory to determinism and implying events that are not caused. Also, in the same article, it says that there is no way to scientifically validate determinism, but it takes a different tack concerning indeterminism. And finally, in the same article, it states that QM (at least according to the Copenhagen Interpretation) is indeterministic.

Stephen Hawking (perhaps the world's most prominent physicist) argues that "scientific determinism" is define in terms of the philosophical notion of "causal determinism" (the view that all events have a cause and effect). See Wikipedia article "Scientific determinism."

nigelTheBold wrote:
Scientists are not in agreement on this subject.

Agreed. I was referring to the Copenhagen Interpretation (which is considered to be the standard interpretation and as we have agreed is not really an interpretation at all but merely a description of what is being observed). Also, there only three (out of thirteen) interpretations that are deterministic: MWI (many worlds), MMI (many minds), and the Bohm interpretation.

nigelTheBold wrote:
For instance, the quantum decoherence crowd believes that every possible superposition is realized, resulting in the "multi-universe" concept. This essentially means that each instance of the superposition isn't random, but inevitable.

Yes, actually it invokes the MWI to eliminate the indeterministic element. Why? Because "uncaused events" are logically unacceptable to most scientists. Also, the MWI is really a metaphysical belief (not a scientific hypothesis). And the MMI is just an logical extension of the MWI. And factor "quantum immortality" into the mix and we have a worldview which supports the idea of a "conscious multi-verse" which implies some type of pantheism or panentheism (i.e. some kind of God-concept). (You can find the same worldview in the "Seth material" written by Jane Roberts who channelled a disincarnate entity named "Seth" )

nigelTheBold wrote:
Another example: Gerard 't Hooft proposes a model which is strictly chaotic, in which particles and their properties are composed of planck-scale artefacts.

But this is at best only a hypothesis, not a validated scientific theory.

nigelTheBold wrote:
Then there's your assertion that the Copenhagen Interpretation is non-causal:

John G. Cramer, University of Washington wrote:
While this element of the [Copenhagen Interpretation] may not satisfy the desires of some physicists for a completely predictive and deterministic theory, it must be considered as at least an adequate solution to the problem unless a better alternative can be found. Perhaps the greatest weakness of [the Copenhagen Interpretation] in this context is not that it asserts an intrinsic randomness but that it supplies no insight into the nature or origin of this randomness.

The Copenhagen Interpretation specifies only that the system is statistical in nature, not that it is uncaused. The CI specifically does not provide "insight into the nature" of the randomness.

You didn't provide the source to the above quote; therefore, I'm unable to read it in context. That being said, I don't see what you think the quote from John G. Cramer is proving. It is true that the CI does not supply "insight into the nature or origin of this randomness." Truly random events are without cause and therefore the CI does not speculate upon what is the nature of this. Science is not in the business of metaphysical speculation. It simply reports what is being observed. This is why the Copenhagen Interpretation is not really an interpretation. All other interpretations are either sceintific or metaphysical speculations.

Also, I did a little research on John G. Cramer. Apparently he is conducting experiments on "retrocausality" - reverse causality or backwards caustion (i.e. the belief that future events can effect the past and present). His interest in this is based on QM. Obviously, he's dabbling in the metaphysical realm of time travel.

nigelTheBold wrote:
Paisley wrote:
I am fully aware that probability theory has numerous applications and is used to describe processes that, for all intents and practical purposes, are considered to be random. However, probabilities in the context of QM is being used to describe a phenomenon that is believed to be a "pure chance" event. This is what you're not getting!

That has not even remotely been established. Not by you, not by any physicist. Not by anyone. That's what you're not getting.

I have said repeatedly in various threads that the prevailing scientific evidence indicates that nature is fundamentally indeterministic and that there is evidence for some events (i.e. quantum events) occurring without cause.This is a true and accurate statement. If you disagree, then please provide me with evidence of what is the cause of quantum events. Note, I said evidence not speculations.

nigelTheBold wrote:
Physicists studying QM can't even agree on the nature of the randomness, as I hope I have successfully established. Your constant assertions that they have is inconsistent with reality.

No, you haven't established that. Scientists know what indeterministic means. It means that there are some events "without cause" and is contradictory to determinism (which means that every event has a cause). I'm sure that Einstein understood this. And I suspect this is one of the reasons why he had a major issue with QM and the Copenhagen Interpretation.

nigelTheBold wrote:
One of the reasons I gave you my "resume" was to provoke you into revealing you are a philosopher, most likely a philosophy professor. In my experience, they are the only ones who insist (as you do) that the "uncaused events" is the sole possible interpretation of Quantum Mechanics. Although physicists often have their own beliefs (such as the many-worlds interpretation of quantum decoherence), most will admit that they have no proof (yet), and that other possibilities (espoused by other physicists) are also likely.

I have never professed to be in academia. If I were, then I wouldn't have the time to debate on internet forums such as this one. Also, you're argument against philosophers is a bogus one. You do realize that "philosophy of science" is a sub-discipline in philosophy...right? Also, there are many philosophers in academia who have a background in physics and mathematics. So, I think you may be selling academic philosophers a little short.

By the way, I have never stated that quantum indeterminism is the only interpretation of QM.

nigelTheBold wrote:
I agree that uncaused events is one possible interpretation. I get that. However, it's not the only possible interpretation; nor is it the one that is most accepted. (As I pointed out, the CI does not specify whether the randomness is "caused" or "uncaused." )

The CI does specify quantum indeterminacy. The "uncauased" element is implicit in the definition of indeterminism. If quantum events have a cause, then CI certainly does not specify it. To date, there is no known cause of quantum events. This is a fact. And it is indisputable. Any talk about causes in relation to quantum events is mere speculation. You know that and I know that.

nigelTheBodl wrote:
Therefore, you can talk of probabilities, and of potential consequences. However, you cannot state categorically that quantum events are uncaused and remain in the realm of the current ontology.

Now, our problem may be that we are simply quibbling over the meaning of "causal" events. But I don't think so, as you seem to get that chaos and "pure chance" are distinct.

Yes, I do understand that the term "chaos" in the context of chaos theory does not have same meaning that is conventionally associated with it. But the same does not apply to term "indeterminism" in relation to QM. You seem to be saying that if something is "unpredictable" then it is indeterminate (in the scientific sense). However, if this were true, then why is chaos theory said to be deterministic when it is also said to be unpredictable?

nigelTheBodl wrote:
Paisley wrote:
Previously, you described theses hypotheses with an acronym - I believe the term you used was "SWAG" or something like that. Can you please define what the acronym stands for. I can't remember. Thank you.

A scientific hypothesis must be testable and falsifiable.

"Scientific Wild-Assed Guess."

Then I guess those QM interpretations that you call scientific "hypotheses" are really SWAGs.

nigelTheBodl wrote:
Paisley wrote:
The many-worlds interpretation (MWI) is actually an interpretation that is held by most scientists (a whopping 60%).
http://en.wikipedia.org/wiki/Many-worlds_interpretation
Do you believe that MWI is testable and falsifiable? Please explain to me how 60% of scientists believe in an infinite number of parallel universes for which there is absolutely no scientific proof? I would say this smacks of metaphysics. What do you think?

It definitely smacks of metaphysics. I agree. It's an outcome that is predicted by quantum decoherence, but there's no way to access data on whether or not these "other worlds" even exist.
Definitely sounds like metaphysics to me.

Good! You agree with me. You're so much more likeable when you're agreeable.

nigelTheBold wrote:
Bell's theorem rules out local variables. It doesn't rule out global variables. (Planck-scale energy variances, the same phenomena that potentially gives rise to virtual particles.)
 

I can't comment of this. I not familiar with the implications of global variables. Are you implying Bell's theorem does not apply in this situation and that chaos theory can solve all the so-called weirdness associated with QM , even "nonlocality?"

"Scientists animated by the purpose of proving they are purposeless constitute an interesting subject for study." - Alfred North Whitehead


Eloise
TheistBronze Member
Eloise's picture
Posts: 1808
Joined: 2007-05-26
User is offlineOffline
Paisley wrote:nigelTheBold

Paisley wrote:

nigelTheBold wrote:
Paisley wrote:

I suspect that the connection between chaos theory and quantum mechanics may be to provide an explanation how small perturbations (random quantum fluctuations) in the microworld (i.e. quantum events) can have a major effect in the macroworld. The argument is sometimes made that quantum events are only relevant in the microworld of subatomic particles, not in the macroworld of Newtonian physics. Chaos theory can provide a basis to explain why this is not so and that the macroworld may also be indeterminate.

On that, we definitely agree. (Except, I suspect, for our definition of "indeterminate." )

Good. But please explain the difference between a theory that is...

1) unpredictable and yet deterministic (e.g. chaos theory), and one that is...

2) unpredictable and indeterministic (e.g. quantum theory)

I told you this one Paisley. You cannot characterise quantum theory itself as non-deterministic, the formalism of the theory can be taken to infer that events have no preceding causes if that formalism is considered with a certain eye but there is no more evidence for this conclusion than there is for other relevant deterministic conclusions (ie Bohm is still kicking regardless of how you dismiss it, and the Everettian version is as brilliant as ever it was.)

Paisley wrote:

 

nigelTheBold wrote:
For instance, the quantum decoherence crowd believes that every possible superposition is realized, resulting in the "multi-universe" concept. This essentially means that each instance of the superposition isn't random, but inevitable.

Yes, actually it invokes the MWI to eliminate the indeterministic element. Why? Because "uncaused events" are logically unacceptable to most scientists. Also, the MWI is really a metaphysical belief (not a scientific hypothesis).

This is a common misconception about MWI, and allow me to set you straight.  When you don't 'measure' the path of a photon it takes every path and leaves concrete evidence of having literally done so - This is called an interference pattern. Quantum interference patterns are formally the result of superposition states interacting with each other and in order to interact with each other each superposition state must somehow EXIST. Hence the foundation of the many worlds interpretation - IF this thing that does exist (important to note) exists universally also (which it should do, we have no evidence or concrete reason to suspect that it doesn't) then there is a multiverse. This is not a metaphysical world view, Paisley, this is a scientific hypothesis. Interference EXISTS, what does it exist as if not exactly what the wave equation supplies formally THAT it exists as? ie copies of the universe.

There are other scientific hypotheses that end up with MWI, but the good thing about this one is that it is so easy to show how scientific it is.

MWI simply states -

!. A exists (known)

2. A exists as the entity the relevant model predicts.

 

Theist badge qualifier : Gnostic/Philosophical Panentheist

www.mathematicianspictures.com


Eloise
TheistBronze Member
Eloise's picture
Posts: 1808
Joined: 2007-05-26
User is offlineOffline
Paisley wrote:Also, I did a

Paisley wrote:

Also, I did a little research on John G. Cramer. Apparently he is conducting experiments on "retrocausality" - reverse causality or backwards caustion (i.e. the belief that future events can effect the past and present). His interest in this is based on QM. Obviously, he's dabbling in the metaphysical realm of time travel.

Oh I didn't see this before, may I suggest you read something on Hawking/Hertog 's top-down cosmology to assuage you of this misconception that retrocausality is a metaphysical dabbling interest in time travel.

And this....

 

Paisley wrote:

nigelTheBold wrote:
Bell's theorem rules out local variables. It doesn't rule out global variables. (Planck-scale energy variances, the same phenomena that potentially gives rise to virtual particles.)
 

I can't comment of this. I not familiar with the implications of global variables. Are you implying Bell's theorem does not apply in this situation and that chaos theory can solve all the so-called weirdness associated with QM , even "nonlocality?"

It means violating Bell doesn't hurt the Bohm interpretation which has global variables (or Everett who never challenges non-locality in any direct way). Chaos theory needs local variables, though, so Chaos, unless one is willing to refute the evidence from Bell-type experiments, doesn't solve quantum indeterminacy.

That said, I might note that this doesn't negate BobSpence1's argument because he only referred to Chaos as an example of a probabilistic/ and for all intents and purposes indeterminable system that involves causality.

 

Theist badge qualifier : Gnostic/Philosophical Panentheist

www.mathematicianspictures.com


Paisley
Theist
Paisley's picture
Posts: 1933
Joined: 2008-03-13
User is offlineOffline
Eloise wrote:Paisley

Eloise wrote:
Paisley wrote:

The many-worlds interpretation (MWI) is actually an interpretation that is held by most scientists (a whopping 60%).

http://en.wikipedia.org/wiki/Many-worlds_interpretation

Do you believe that MWI is testable and falsifiable? Please explain to me how 60% of scientists believe in an infinite number of parallel universes for which there is absolutely no scientific proof? I would say this smacks of metaphysics. What do you think?

Ok stop right there, this is ludicrous, you're arguing the collapse postulate and telling us you reject MWI on lack of scientific evidence? I take it you can prove the collapse postulate then? Falsify interference at size levels above the buckyball? There is no more evidence for collapse than there is for no collapse, Paisley, so please get your facts straight before you shoot low.

I'll have you know the relative state formulation is rigorously mathematical (identical to other formulations of QM in this sense) and thoroughly well supported by many of the best and most noteworthy advances in quantum cosmology and quantum technology in the last decade (in all other respects). To swipe a clever retort from DG there are a great many things that just cannot make sense except in the context of the theory of all quantum states being actualised. 60% of physicists (is it really that high these days? Wow!) believe it because so consistently are successful advances in the theory tracing back to it in one form or another. 

Oh and one more thing: this time you can rightly suppose that you have learned something about what I believe, because this time I am arguing my position, not someone else's.

PS: If you are going to insist that no collapse is not scientifically verified while simultaneously arguing that the collapse postulate is absolute truth, then explain how the quantum eraser experiment can restore the wavefunction post perturbation while you're at it.

Huh? What is this all about?

To begin with, I have never stated that I believe that the CCC is the only correct interpretation of QM. In fact, I have never argued for the CCC. I don't know where you are getting this and why you are accusing me of this. That being said, I do believe that conscious-awareness is fundamental and I have publicly stated this. But this is not because of any QM interpretation commitment. My religious or spiritual beliefs are not founded on QM. It's based on faith or what I call a basic spiritual intuition.

Secondly, there are aspects of various QM interpretations that I like. However, I have not studied every QM interpretation that has been formulated. Therefore, I am not committed to any particular one. 

Thirdly, I was not arguing against the MWI per se. (Actually, the idea of many worlds appeals to my sensibilities.) I was simply suggesting that it was really a metaphysical position and not a bonafide scientific hypothesis (i.e. it's not really testable or falsifiable). No doubt, it is a metaphysical position that is grounded in rigorous mathematics - mathematics that is way beyond my level of understanding; nevertheless, it's only a metaphysical position. And I might add that Nigel agrees with me on this.

Fourthly, I am not against metaphysical belief - after all, I have openly professed belief in God. The only reason I am making the argument that MWI is a metaphysical belief is that atheists characterize belief in God as being irrational (at least the atheist materialists who subscribe to logical positivism...I will assume that both Nigel and Bob Spence fall into this category).  My argument is basically this: The atheist materialist who subcribes to logical positivism cannot subscribe to metaphysical beliefs when their philosophical commitment doesn't allow it. Certainly, the atheist cannot say that belief in God is irrational while permitting himself the luxury of believing in a multi-verse. This is what is ridiculous. They can't have it both ways. And I'm simply letting them know.

Fifthly, why are you taking this hissy fit about RQM? I have never said anything specifically against RQM. I understand that RQM is related to the MWI and that you may be somewhat offended that I called MWI metaphysics. But, as I said previously, I'm actually inclined to believe in parallel worlds. And I certainly have a relational or process worldview. In fact, I recently completed Alan B. Wallace's book entitled "Hidden Dimensions." He has a background in physics, philosophy of science, and Buddhism. In his book, Wallace presents a theory he calls the "special and general theory of ontological relativity." I found it very appealing and it seems to mesh with RQM. So, I'm not really attacking RQM.

Finally, I am happy to see that you are voicing your views. However, what are you really voicing. Are you making an argument for some kind of theistic or spiritual belief? Are you challenging atheistic materialism? Are you explaining how RQM relates to panentheism? If you are, I haven't seen it yet.

"Scientists animated by the purpose of proving they are purposeless constitute an interesting subject for study." - Alfred North Whitehead


I AM GOD AS YOU
Superfan
Posts: 4793
Joined: 2007-09-29
User is offlineOffline
Pantheists with SWORDS ! Yes

Pantheists with SWORDS ! Yes Yes debate .... 

                               no more "E I S T S"    

            no more A T P B ............. no separation   /  ONE 

                           no more arguing the why (dogma - religion)

                                            ASK HOW

                                                       SCIENCE

                                                                               * 


Eloise
TheistBronze Member
Eloise's picture
Posts: 1808
Joined: 2007-05-26
User is offlineOffline
Paisley wrote:Eloise

Paisley wrote:

 

Eloise wrote:
Paisley wrote:

 

The many-worlds interpretation (MWI) is actually an interpretation that is held by most scientists (a whopping 60%).

 

http://en.wikipedia.org/wiki/Many-worlds_interpretation

 

Do you believe that MWI is testable and falsifiable? Please explain to me how 60% of scientists believe in an infinite number of parallel universes for which there is absolutely no scientific proof? I would say this smacks of metaphysics. What do you think?

 

 

Ok stop right there, this is ludicrous, you're arguing the collapse postulate and telling us you reject MWI on lack of scientific evidence? I take it you can prove the collapse postulate then? Falsify interference at size levels above the buckyball? There is no more evidence for collapse than there is for no collapse, Paisley, so please get your facts straight before you shoot low.

 

I'll have you know the relative state formulation is rigorously mathematical (identical to other formulations of QM in this sense) and thoroughly well supported by many of the best and most noteworthy advances in quantum cosmology and quantum technology in the last decade (in all other respects). To swipe a clever retort from DG there are a great many things that just cannot make sense except in the context of the theory of all quantum states being actualised. 60% of physicists (is it really that high these days? Wow!) believe it because so consistently are successful advances in the theory tracing back to it in one form or another.

 

Oh and one more thing: this time you can rightly suppose that you have learned something about what I believe, because this time I am arguing my position, not someone else's.

 

PS: If you are going to insist that no collapse is not scientifically verified while simultaneously arguing that the collapse postulate is absolute truth, then explain how the quantum eraser experiment can restore the wavefunction post perturbation while you're at it.

 

Huh? What is this all about?

 

To begin with, I have never stated that I believe that the CCC is the only correct interpretation of QM. In fact, I have never argued for the CCC. I don't know where you are getting this and why you are accusing me of this.

 

 

I explained this to you in the other thread, your argument for absolute fundamental indeterminacy explained by the caprice of a conscious will is precisely a CCC type theory, the only way it is possible that an 'elemental mind' can determine a unique outcome of a quantum measurement is via a post-measurement projection from the pure state into the measured state which prescribes that the rest of the wavefunction 'collapse'. Thus what you have stated is that you do favour CCC.

 

I have no objection to this in itself, you can favour any explanation you want and build whatever metaphysic takes your fancy from it, given the circumstances this is all quite reasonable in my estimation. Where you were mistaken most greatly was in then stating that a no collapse interpretation was without evidence, I'm afraid you forfeited the luxury of saying that when you chose to favour collapse with equally negligible evidence. You see?


 

Paisley wrote:

 

Thirdly, I was not arguing against the MWI per se. (Actually, the idea of many worlds appeals to my sensibilities.) I was simply suggesting that it was really a metaphysical position and not a bonafide scientific hypothesis (i.e. it's not really testable or falsifiable). No doubt, it is a metaphysical position that is grounded in rigorous mathematics - mathematics that is way beyond my level of understanding; nevertheless, it's only a metaphysical position.

 

 

The thing to note here is that many worlds is not merely a metaphysical position, at least it's not any more than saying 'colour is a wavelength of light and not a thing in itself' is a metaphysical position. That is to say, it's actually not really metaphysical at all, it is a very faithful analogue of formal Quantum mechanics. I hope I'll be able to clarify this for you.


 

Paisley wrote:

 

Fourthly, I am not against metaphysical belief - after all, I have openly professed belief in God. The only reason I am making the argument that MWI is a metaphysical belief is that atheists characterize belief in God as being irrational (at least the atheist materialists who subscribe to logical positivism...I will assume that both Nigel and Bob Spence fall into this category).  

 

 

I know you're not against a metaphysical belief, that is not why I argued back about MWI. When I say that MWI is mischaracterised by the word metaphysical it's because I mean it's mischaracterised by the word metaphysical, not because I take offense to the idea of a metaphysical position.

 

Okay, so the idea of parallel universe reeks of metaphysics, I concede that much but the MWI itself does not speculate that many worlds underlie the quantum phenomenon, it simple follows quantum formalism directly, avoiding assumptions, to that conclusion.

 

I understand what you are saying about the logical positivist line that god belief is irrational and I empathise with your feelings toward it. That said, it doesn't bother me much as I am far more interested in the general advancement of human knowledge than in getting heated over a small slight that is actually quite true in a great many cases.


 

Paisley wrote:

 

Fifthly, why are you taking this hissy fit about RQM? I have never said anything specifically against RQM.

 

 

Well clearly, then, it was inadvertant so I apologise for the hissy fit.


 

Paisley wrote:

 

Finally, I am happy to see that you are voicing your views. However, what are you really voicing. Are you making an argument for some kind of theistic or spiritual belief? Are you challenging atheistic materialism? Are you explaining how RQM relates to panentheism? If you are, I haven't seen it yet.

 

 

Here I was addressing a frustrating misconception about what it means to think the multiverse is a true theory, so no, it's not meant to say anything in particular about how RQM relates to panentheism, only, obviously, that it does in regards to my beliefs.

Theist badge qualifier : Gnostic/Philosophical Panentheist

www.mathematicianspictures.com


I AM GOD AS YOU
Superfan
Posts: 4793
Joined: 2007-09-29
User is offlineOffline
I AM GOD AS YOU wrote:All

I AM GOD AS YOU wrote:

All religiously related "isms" are ultimately "absurd". (borrowing 'absurd' from P )

 [Edit my post 58 - addition 6/13]   

Pantheism must also eventually die , I ain't going for it, I AM in a hurry. I envision a future where Atheism will also have no philosophical use, as the very word "god" (the force) will be universally understood as 100% atheist ! All is ONE.

The science questions of QM and consciousness ( C ) will have no need for "theology" nor  "isms" .....

  The placement of C regarding E/M regarding the "force" will be better understood.

  I cannot imagine C pre-existing E/M , but it is not of any major philosophical problem for me. All is ONE , connected, regardless of the ordering arrangement of the parts  F  C  E/M  etc ......

I ask the pantheists, and panentheists not to fuel the dogma of division as is the God of Abe doctrine.

Pantheism is C science philosophy. Thanks, igod as YOU. We are the "Force"!  

  


QuasarX
QuasarX's picture
Posts: 242
Joined: 2007-10-04
User is offlineOffline
Hambydammit wrote:That means

Hambydammit wrote:
That means that a thing is presumed false until proven true.

I used to do that also, but I got tired of finding out that things I presumed false were actually true.  Now, I instead presume that a thing is unknown until shown to be either true or false.  It seems, however, that the bright folks who brought us Battleground God (http://www.philosophersnet.com/games/god.htm) think that this would require me to consider all sorts of wild possibilities, like pink elephants living on the surface of Venus.  This is not true, however!  I can simply say that I don't care whether or not there are pink elephants living on the surface of Venus and go about my business.


Eloise
TheistBronze Member
Eloise's picture
Posts: 1808
Joined: 2007-05-26
User is offlineOffline
QuasarX wrote:Hambydammit

QuasarX wrote:

Hambydammit wrote:
That means that a thing is presumed false until proven true.

I used to do that also, but I got tired of finding out that things I presumed false were actually true.  Now, I instead presume that a thing is unknown until shown to be either true or false.  It seems, however, that the bright folks who brought us Battleground God (http://www.philosophersnet.com/games/god.htm) think that this would require me to consider all sorts of wild possibilities, like pink elephants living on the surface of Venus.  This is not true, however!  I can simply say that I don't care whether or not there are pink elephants living on the surface of Venus and go about my business.

Hi Quasar, nice to see you around again. I took that test just now, I took no hits and bit one bullet at question 16 - basically by admitting when challenged that my concept of God doesn't honour the laws of identity and excluded middle so faithfully, which is true and I have been open about that here. Anyhow, excellent link. Thanks.

EDIT: I just check the FAQ to see if my objection to the bullet was covered and this is their reply to my objection:

b) It isn't straightforwardly the case that propositions which are contradictory or absurd are meaningless. If that is your claim, then it needs to be grounded in a proper theory of meaning;

Fine with me, I have the multiverse to provide that.

 

Theist badge qualifier : Gnostic/Philosophical Panentheist

www.mathematicianspictures.com


I AM GOD AS YOU
Superfan
Posts: 4793
Joined: 2007-09-29
User is offlineOffline
Hi Eloise. I Bit one Bullet

God test: http://www.philosophersnet.com/games/god.htm 


Hi Eloise. I Bit one Bullet too , so they say, quote;

You answered "True" to questions 6 and 13.

These answers generated the following response:

You stated earlier that evolutionary theory is essentially true. However, you have now claimed that it is foolish to believe in God without certain, irrevocable proof that she exists. The problem is that there is no certain proof that evolutionary theory is true - even though there is overwhelming evidence that it is true. So it seems that you require certain, irrevocable proof for God's existence, but accept evolutionary theory without certain proof. So you've got a choice: (a) Bite a bullet and claim that a higher standard of proof is required for belief in God than for belief in evolution. (b) Take a hit, conceding that there is a contradiction in your responses.

You chose to bite the bullet.  /////
_____________________________

No, you gave me only 2 options. You bite the bullet. Do I need proof I exist? Evolution basics is 100%  proven, not how life began etc.

Damn fucking nitpickers !  LOL ....   

I also think a "God definition" needs to be stated before the test. igod !

_____________________________

A question regarding the "Force": What came first Consciousness or E/M ?  Go philosophy science. Buddha answers, whatever, "don't "worry" about it, and please no dogma".  

I say it's ONE connected damn interesting "vibration" ! Let's party to the vibration ! 

    Seriously tho, seems C is a product of E/M , not the other way around. Just my lame guess     What Am I , (god of course), but what is that ?  

____________________________

For new readers, basics about Pantheism, Panentheism. Any objections Eloise etc to this wiki explanation ?

http://en.wikipedia.org/wiki/Panentheism

 


Eloise
TheistBronze Member
Eloise's picture
Posts: 1808
Joined: 2007-05-26
User is offlineOffline
I AM GOD AS YOU wrote:God

I AM GOD AS YOU wrote:

God test: http://www.philosophersnet.com/games/god.htm 

 

Hi Eloise. I Bit one Bullet too , so they say, quote;

You answered "True" to questions 6 and 13.

These answers generated the following response:

You stated earlier that evolutionary theory is essentially true. However, you have now claimed that it is foolish to believe in God without certain, irrevocable proof that she exists. The problem is that there is no certain proof that evolutionary theory is true - even though there is overwhelming evidence that it is true. So it seems that you require certain, irrevocable proof for God's existence, but accept evolutionary theory without certain proof. So you've got a choice: (a) Bite a bullet and claim that a higher standard of proof is required for belief in God than for belief in evolution. (b) Take a hit, conceding that there is a contradiction in your responses.

You chose to bite the bullet.  /////
_____________________________

No, you gave me only 2 options. You bite the bullet. Do I need proof I exist? Evolution basics is 100%  proven, not how life began etc.

Damn fucking nitpickers !  LOL ....   

I also think a "God definition" needs to be stated before the test. igod !

_____________________________

 

Hi Iam,

Biting the bullet is not a logical error; the bullet you bite is a bullet you shot earlier - in your case your answer to number 6 came back to bite you at number 13. At this point you can let it bite you, or you can bite it. If you bite the bullet basically it means that you are prepared to stick by your first conviction, your first answer. if you don't stick by your first answer then you are hit by a logical inconsistency.

It's perfectly alright to bite the bullet as long as you're willing to back up your bravado with some justifying statement. In your case, as the creators of the site pointed out, biting the bullet could mean admitting that you think God needs to comply with a higher standard of proof than evolution.

Remember You CAN say that you expect more from God than from evolution in terms of proof, you know, it's quite justifiable in plenty of ways for example which one of these is conventionally capitalised? and... which one demands that you surrender your mind, body and soul unquestioningly to it?

The main thing about biting the bullet is to be aware that you are doing it and avoid the logical contradiction that you may be setting yourself up for.

PS: I'll check out the other link for you later gotta go.

Theist badge qualifier : Gnostic/Philosophical Panentheist

www.mathematicianspictures.com


I AM GOD AS YOU
Superfan
Posts: 4793
Joined: 2007-09-29
User is offlineOffline
Thanks Eloise. Ahhh, "biting

Thanks Eloise. Ahhh, "biting the bullet". I get it. Ha ha, I do alot of that, claiming you and I and everything is god ! You are a kind teacher ..... pretty too ....

Thanks for putting up with the theist badge. I really think RRS should include a "Pantheist" badge, because that non dogmatic God philosophy message is helpful to the people .... 

Hey girl, you is a "sexed up atheist" ! (Richard Dawkins)    You rock E !   LOL 

 


QuasarX
QuasarX's picture
Posts: 242
Joined: 2007-10-04
User is offlineOffline
Hi Eloise.  I wish I could

Hi Eloise.  I wish I could post here more often, but life keeps getting in the way.  I also take no hits and bite 1 bullet when I answer in accordance with my current world view.  The bullet I bite is that I don't accept absence of evidence as evidence of absence, which seems to be the logical high ground from my point of view... less likely to lead to errors/false beliefs.  I should probably start a thread about that sometime.  And yes, their FAQ seems to address a lot of common justifications for bullet-biting... in some ways I consider reading it to be more valuable than taking the quiz (though I highly recommend taking the quiz before reading the FAQ).


Thomathy
Superfan
Thomathy's picture
Posts: 1861
Joined: 2007-08-20
User is offlineOffline
QuasarX wrote:Hambydammit

QuasarX wrote:

Hambydammit wrote:
That means that a thing is presumed false until proven true.

I used to do that also, but I got tired of finding out that things I presumed false were actually true.  Now, I instead presume that a thing is unknown until shown to be either true or false.  It seems, however, that the bright folks who brought us Battleground God (http://www.philosophersnet.com/games/god.htm) think that this would require me to consider all sorts of wild possibilities, like pink elephants living on the surface of Venus.  This is not true, however!  I can simply say that I don't care whether or not there are pink elephants living on the surface of Venus and go about my business.

I took one bullet.  In the question regarding the proof for god I did not put it in the same category as the evidence that supports evolution.  I think that evolution and the proposition of god are categorically different.  I didn't bite the bullet in case I was actually wrong... I really don't think I was, but someone can tell me if indeed I am.  Evolution is a theory that is built upon laws and massive amounts of evidence.  A theory can necessarily be altered by new evidence in order to maintain its descriptive abilities.  God, on the other hand, exists or does not, the proposition is not a theory.  There is only one required piece of evidence to prove the existence of god, for god to tell me he exists -and that's a part from the other evidence I'd accept to make me really suspicious or even to make me believe.

BigUniverse wrote,

"Well the things that happen less often are more likely to be the result of the supper natural. A thing like loosing my keys in the morning is not likely supper natural, but finding a thousand dollars or meeting a celebrity might be."


QuasarX
QuasarX's picture
Posts: 242
Joined: 2007-10-04
User is offlineOffline
Thomathy wrote:I took one

Thomathy wrote:
I took one bullet.  In the question regarding the proof for god I did not put it in the same category as the evidence that supports evolution.  I think that evolution and the proposition of god are categorically different.  I didn't bite the bullet in case I was actually wrong... I really don't think I was, but someone can tell me if indeed I am.  Evolution is a theory that is built upon laws and massive amounts of evidence.  A theory can necessarily be altered by new evidence in order to maintain its descriptive abilities.  God, on the other hand, exists or does not, the proposition is not a theory.  There is only one required piece of evidence to prove the existence of god, for god to tell me he exists -and that's a part from the other evidence I'd accept to make me really suspicious or even to make me believe.

Hmm... well, I agree there are differences between god-belief and belief in evolution.  Although, whether they admit it or not, Christians modify their collective belief in God all the time, sometimes to adapt to scientific evidence, and other times to try to rationalize away logical contradictions.  That's why there are so many different versions of Christianity in existence.  I believe the same thing happens to other religions as well, including Islam, Buddhism, and probably others that I haven't really paid attention to.  "God" may not be alterable, but people's ideas of "God" certainly are.  Similarly, the mechanics of evolution exist as they do, regardless of what people say they are... but people can adapt their ideas of what evolution is to fit scientific data or religious faith, which is how we get arguments and court cases over evolution and intelligent design.

I personally don't distinguish between one type of belief and another, because every belief is based on something, and most beliefs are held without sufficient evidence to justify certainty.  Well, that and the fact that I prefer rule systems to be as simple and straightforward as can be practically accomplished.  That's why I don't inherently object to the belief in some godlike thing... my objections are specific to anti-human god beliefs like Christianity and to attempts to force others to accept beliefs which aren't supported by evidence.


MattShizzle
Posts: 7966
Joined: 2006-03-31
User is offlineOffline
I bit one bullet on saying

I bit one bullet on saying if god exists he should be able to create square circles or make 1 + 1 = 17. Well, to me anything else wouldn't be omniscience.

Matt Shizzle has been banned from the Rational Response Squad website. This event shall provide an atmosphere more conducive to social growth. - Majority of the mod team


QuasarX
QuasarX's picture
Posts: 242
Joined: 2007-10-04
User is offlineOffline
I'm guessing you meant

I'm guessing you meant omnipotence, Matt?  But, you have a valid point.  If someone has the power to do anything, that should include the power to change the fundamental laws of the universe, right?